LSAT and Law School Admissions Forum

Get expert LSAT preparation and law school admissions advice from PowerScore Test Preparation.

 Administrator
PowerScore Staff
  • PowerScore Staff
  • Posts: 8916
  • Joined: Feb 02, 2011
|
#25330
Complete Question Explanation
(See the complete passage discussion here: lsat/viewtopic.php?t=10362)

The correct answer choice is (C)

When answering an open-ended Must Be True—EXCEPT question without a textual referent, prephrasing the answer choice is near impossible. Instead, eliminate any answer choice you can confirm by the information provided in the passage. The answer choice that cannot be proven will be correct.

As you analyze each answer choice, you may notice that they all test your understanding of criminal behavior as a rational economic choice. The last paragraph of the passage, and especially lines 35-40, will be particularly useful in evaluating the author’s position regarding each statement.

Answer choice (A): The author would agree that increased deterrence is likely to result in a reduced crime rate. This answer choice is incorrect.

Answer choice (B): Ending efforts to rehabilitate criminals will decrease the expected utility from lawful activity. Consequently, the rate at which criminals return to criminal activity is likely to increase. Since the author is likely to agree with this statement, answer choice (B) is incorrect.

Answer choice (C): This is the correct answer choice. If the expected utility of lawful activity decreases, it is reasonable to expect that the rate of deliberate crimes will increase, not decrease. Since the author is unlikely to agree with this statement, answer choice (C) is correct.

Answer choice (D): Decreasing access to economic institutions is likely to decrease the economic opportunities of individuals most alienated from such institutions. By lowering the expected utility from lawful activity, this will lead to an increase in the rate of deliberate crimes. Since the author is likely to agree with this statement, answer choice (D) is incorrect.

Answer choice (E): The author is likely to agree that the rate of deliberate crimes is inversely associated with the level of law enforcement: increasing the risk of getting caught is a deterrent that will probably lower the expected utility from criminal activity, thereby making a person less likely to choose to commit a deliberate crime.

Get the most out of your LSAT Prep Plus subscription.

Analyze and track your performance with our Testing and Analytics Package.